#30 Rosh Review

अब Quizwiz के साथ अपने होमवर्क और परीक्षाओं को एस करें!

Question: What substance must be avoided during metronidazole therapy?

Answer: Alcohol. Rapid Review Bacterial Vaginosis Patient will be complaining of malodorous vaginal discharge PE will show thin, gray/white discharge Labs will show pH > 4.5, clue cells Diagnosis is made by KOH to smear → fishy odor, "whiff test", Amsel Criteria Most commonly caused by Gardnerella vaginalis Treatment is metronidazole

Question: What are the 3 types of rectal prolapse?

Answer: Rectal mucosa only (< 2 yrs old); all layers of rectum; intussusception through rectum. Rapid Review Rectal Prolapse (Procidentia) Precipitated by defecating, coughing, and sneezing No vascular compromise: manual reduction in ED Vascular compromise: emergency surgery consultation and reduction Consider CF in pediatric patient Types: Mucosal prolapse (hemorrhoids and children) All layers prolapse Intussusception

Question: Name some neuromuscular causes of childhood secondary scoliosis?

Answer: Spina bifida, cerebral palsy, tethered cord syndrome, muscular dystrophy and spinal muscular atrophy. Rapid Review Scoliosis Lateral curvature of the spine >10 degrees F > M Causes: Idiopathic > neuromuscular disease Forward bending test: assesses vertebral and rib rotation Cobb angle: quantifies magnitude of deformity

Question: What is the oral chelation medication for lead poisoning?

Answer: Succimer. Rapid Review Lead Poisoning Joint pain, constipation Lead lines: hyperdense lines at metaphyses Microcytic hypochromic anemia Basophilic stippling Oral succimer, IV EDTA

Question: How long does it take for delirium to fully resolve?

Answer: It may take weeks to months for delirium to fully resolve. Rapid Review Delirium Sudden onset, fluctuation Impaired speech Impaired orientation Disorganized thinking Hallucinations

Question: What are the two side effects associated with long term use of flecainide?

Answer: QRS widening and QT prolongation. Rapid Review Antiarrhythmics: Class IC Class I: fast Na+ channel blockers "Can I have fries, please.": flecainide, propafenone ↑ AV node refractory period ↑ PR/QT/QRS

Question: Which is more elevated in alcoholic hepatitis, AST or ALT?

Answer: AST>ALT in alcoholic liver disease, compared to ALT>AST in viral hepatitis. Rapid Review Acute Hepatitis HAV: fecal-oral, shellfish, alone (no carrier), asymptomatic, acute HBV: HBsAg: active infection Anti-HBs: recovered or immunized Anti-HBc IgM: early marker of infection, positive in window period Anti-HBc IgG: best marker for prior HBV HBeAg: high infectivity Anti-HBeAb: low infectivity HCV: IVDA, chronic, cirrhosis, carcinoma, carrier HDV: dependent on HBV coinfection HEV: fecal-oral (enteric) high mortality rate among pregnant (expectant) patients, epidemics, HAV and HEV are fecal-oral: "The vowels hit your bowels" Autoimmune hepatitis: young females Alcoholic hepatitis: moderate transaminase elevation, AST>ALT Supportive rx

Question: Which abnormal reflex is most indicative of intraspinal pathology?

Answer: Abdominal.

Question: Name one complication of ventricular septal defect corrective-surgery?

Answer: Conduction defects, such as transient right bundle branch block. Rapid Review Ventricular Septal Defect PE will show a loud, harsh, holosystolic murmur at the lower left sternal border Diagnosis is made by echo Treatment is most close spontaneously Comments: most common pathologic murmur in childhood

Question: What is the characteristic pattern seen when lesions follow the Langer's lines in Pityriasis rosea?

Answer: "Christmas tree" pattern. Rapid Review Pityriasis Rosea Patient with a history of a larger lesion 1 week prior, ​"Herald Patch" Complaining of rash on the back PE will show diffuse papulosquamous rash on the trunk, "Christmas tree-like" distribution Treatment is self-limiting disease, itching with antihistamines

Question: What percentage of patients with posterior vitreous detachment have a retinal detachment?

Answer: 14%. Rapid Review Posterior Vitreous Detachment Precursor to retinal detachment Occurs over weeks to months Associated with floaters

Question: What is a subluxed tooth?

Answer: A tooth loosened in its socket as a result of a force. Rapid Review Tooth Avulsion Replace tooth as soon as possible Primary teeth: do not reimplant Transport in Hank's solution (best), milk Rinse tooth (do not brush)

Question: What is the cellophane tape test for pinworms?

Answer: A two- to three-inch piece of clear tape is applied serially to several perianal areas in the morning before washing. The tape is then applied to a glass slide. Rapid Review Pinworms Transmission via ingestion of Enterobius vermicularis eggs Nocturnal pruritus ani Dx: tape test

Question: What lab marker helps to distinguish primary from secondary adrenal insufficiency?

Answer: An elevated ACTH level in a patient with adrenal insufficiency is consistent with primary adrenal insufficiency. Rapid Review Primary Adrenal Insufficiency (Addison's Disease) Patient will be complaining of abdominal pain, nausea, vomiting, diarrhea, fever, and confusion PE will show hyperpigmentation of skin and mucus membranes and hypotension Labs will show hyponatremia and hyperkalemia Most commonly caused by autoimmune Treatment is hydrocortisone

Question: If a patient has ankle instability, what physical exam tests should be positive?

Answer: Anterior drawer test and talar tilt test. Rapid Review Ankle Sprain Partial or complete tearing of ≥ 1 ligaments MC due to inversion mechanism MC sprained: anterior talofibular ligament (ATFL) Grade I: microscopic tear, grade II: incomplete tear, grade III: complete tear RICE

Question: Using the 1990 criteria, how many trigger points must a patient have to meet criteria for the diagnosis of fibromyalgia?

Answer: At least eleven of eighteen symmetrical trigger points. Rapid Review Fibromyalgia Patient will be a woman Complaining of widespread musculoskeletal pain for > 3 months, non-restorative sleep and generalized fatigue PE will show tenderness at > 9 of 18 anatomic sites ("trigger points") Labs will be normal Diagnosis is made clinically Treatment is education, antidepressants, avoid opioids

Question: What is the most common inheritance pattern of dilated cardiomyopathy?

Answer: Autosomal dominant.

Question: What diagnosis is considered in a child with similar symptoms to croup who appears systemically ill?

Answer: Bacterial tracheiitis. Rapid Review Laryngotracheitis (Croup) Patient will be a non-toxic appearing child, 6 months to 3 years old Complaining of URI symptoms with barky, seal-like cough, inspiratory stridor, low-grade fever Labs will show steeple sign on PA view Most commonly caused by Parainfluenza virus Treatment is steroids, aerosolized epinephrine

Question: What history with subconjunctival hemorrhage would prompt investigation into a bleeding diathesis?

Answer: Bilateral or recurrent subconjunctival hemorrhage should have a bleeding diathesis workup. Rapid Review Subconjunctival Hemorrhage Coughing, sneezing, minor trauma Under conjunctiva and stops at limbus Reassurance

Question: What Category is ranitidine?

Answer: Category B. Rapid Review Hyperemesis Gravidarum Peak incidence: weeks 8-12 Weight loss Hypokalemia Ketonemia Rx: IVF with 5% dextrose, antiemetics

Question: What is Charcot's triad and what is its significance?

Answer: Charcot's triad is fever, jaundice and right upper quadrant pain. This triad is suggestive of ascending cholangitis. Rapid Review Cholelithiasis Patient will be an obese woman 40 - 50 years old Complaining of slowly resolving right upper quadrant pain that begins suddenly after eating a fatty or large meal Diagnosis is made by ultrasound Most commonly made of cholesterol Treatment is observation or cholecystectomy Comments: Four "F's": Female, Forty, Fat, Fertile

Question: What is Reynold's pentad?

Answer: Charcot's triad plus confusion and hypotension. Rapid Review Acute Cholangitis Patient will be complaining of right upper quadrant pain, jaundice, fever (Charcot's triad) Diagnosis is made by ERCP Most commonly caused by choledocholithiasis leading to bacterial infection, E.coli Treatment is antibiotics Comments: Charcot's triad + hypotenstion and AMS = Reynold's pentad, acute obstruction

Question: What is the most accurate radiographic test to diagnose acute cholecystitis?

Answer: Cholescintigraphy is the most accurate radiographic test for diagnosing acute cholecystitis. Rapid Review Cholecystitis Patient will be complaining of colicky, steadily increasing RUQ or epigastric pain after eating fatty foods PE will show Murphy's sign, Boas sign Diagnosis is made by: Initial: US, Gold standard: HIDA Most commonly caused by obstruction by a gallstone Treatment is cholecystectomy

Question: Which drug used for the treatment of heart failure has been shown to reduce hospitalizations, but has no effect on mortality?

Answer: Digoxin. Rapid Review Heart Failure Staging/Classification American Heart Association/American College of Cardiology staging: Stage A: high risk without symptoms/disease Stage B: structural disease without sx Stage C: structural disease + sx Stage D: refractory heart failure New York Heart Association classification: I: asymptomatic II: sx with ordinary activity III: asymptomatic only at rest IV: sx at rest

Question: What tapeworm infection is associated with vitamin B12 deficiency?

Answer: Diphyllobathrium latum infection causes vitamin B12 malabsorption. Rapid Review Megaloblastic Anemia B12 deficiency: vegan, pernicious anemia Folate deficiency: alcoholic, antifolate therapy MCV > 100 Hypersegmented neutrophils B12 deficiency: neurologic findings

Question: Following the diagnosis of pertussis, should close contacts be treated?

Answer: Due to the high communicability and potential for severe disease in young children, the CDC recommends initiating antibiotics for all close contacts, regardless of age or vaccination status, preferably within three weeks of diagnosis. Rapid Review Pertussis ("Whooping Cough") Whooping cough: expiratory cough, inspiratory whoop Child with incomplete immunization hx URI sx → paroxysmal stage → convalescence Macrolide

Question: What is considered third-line treatment of continued severe premenstrual symptoms in women who do not tolerate or respond to SSRIs or OCPs?

Answer: Gonadotropin-releasing hormone agonists. Rapid Review Premenstrual Syndrome Onset: 1-2 weeks prior to cycle, resolves after menstruation begins Sleep disturbances ↓ Focus Emotional lability Breast tenderness, HA Does not hinder personal/professional life (unlike premenstrual dysphoric disorder) Initial rx: dec caffeine intake, exercise, stress reduction Pharmacologic rx: NSAIDs, SSRIs, OCPs

Question: How does diabetes mellitus cause polyuria?

Answer: Hyperglycemia spills over into the kidney tubules, pulling water with it (osmotic diuresis). Rapid Review Diabetes Insipidus (DI) ADH deficiency → polyuria + inability to concentrate urine ↑ Plasma osmolality + decreased urine osmolality Central DI: ↓ ADH production Water deprivation test: >50% ↑ in urine osmolality Rx: intranasal DDAVP Nephrogenic DI: Renal unresponsiveness to ADH Water deprivation test: no change in urine osmolality Rx: HCTZ, amiloride, indomethacin

Question: What is the preferred diagnostic study for obstructive sleep apnea?

Answer: In-laboratory polysomnography. Rapid Review Obstructive Sleep Apnea Patient will be obese With a history of allergies Complaining of apneic episodes while sleeping, snoring and daytime sleepiness PE will show enlarged tonsils Diagnosis is made by sleep study Treatment is weight loss, CPAP Comments: can lead to pulmonary hypertension and cor pulmonale (right ventricular hypertrophy)

Question: What is typically seen on histopathology in patients with eosinophilic esophagitis?

Answer: Infiltration of the esophageal squamous epithelium with eosinophils. Additionally, basal cell hyperplasia and lamina propria fibrosis can also seen.

Question: What is the bulbocavernosus reflex?

Answer: Involuntary reflex contraction of the anal sphincter in response to squeezing the glans penis or tugging on the foley catheter. Loss of this reflex is a marker of spinal shock. Rapid Review Anterior Cord Syndrome PE will show loss of motor, pain, and temperature below injury Most commonly caused by flexion injury Comments: proprioception and vibration intact

Question: Why is intravenous triiodothyronine (T3) rarely administered?

Answer: It can precipitate dysrhythmias and sudden death.

Question: What is a Holter monitor?

Answer: It is an ambulatory ECG worn for 24-48 hours and it is the most frequent non-invasive test used to evaluate cardiac rhythm abnormalities.

Question: What findings are seen in keratoconjunctivitis?

Answer: Keratoconjunctivitis is infection of the cornea in addition to the conjunctiva. Findings include eye pain and redness, photophobia, conjunctival injection and diffuse, fine, fluorescein uptake on the cornea. Rapid Review Viral Conjunctivitis Patient will be complaining of bilateral red/pink eye PE will show preauricular lymphadenopathy, copious watery discharge, scant mucoid discharge Most commonly caused by adenovirus Treatment is cool compresses, artificial tears, antihistamines for itching/redness

Question: What is another name for croup?

Answer: Laryngotracheobronchitis. Rapid Review Laryngotracheitis (Croup) Patient will be a non-toxic appearing child, 6 months to 3 years old Complaining of URI symptoms with barky, seal-like cough, inspiratory stridor, low-grade fever Labs will show steeple sign on PA view Most commonly caused by Parainfluenza virus Treatment is steroids, aerosolized epinephrine

Question: What is a common side effect of amlodipine?

Answer: Lower extremity edema. Rapid Review Hypertension: Eighth Joint National Committee (JNC 8) Recommendations PreHTN: systolic blood pressure (SBP) 120-139 mmHg or diastolic blood pressure (DBP) 80-89 mmHg Stage I HTN: SBP 140-159 mmHg or DBP 90-99 mmHg Stage II HTN: SBP >160 mmHg or DBP >100 mmHg Treatment goals: >60 years: SBP <150, DBP <90 All others: SBP <140, DBP <90 1st line rx for general population: thiazide, CCB, ACEI, or ARB 1st line rx for African Americans: CCB or thiazide Chronic kidney disease: Rx should include ACEI or ARB

Question: What imaging modality should be used to evaluate a patient for vertebrobasilar insufficiency?

Answer: MRI of the brain with MRA. Rapid Review Dix-Hallpike Maneuver Evaluates BPPV Positive test: symptoms, nystagmus

Question: What is the most common cause of a massive pleural effusion (> 1.5 - 2 L)?

Answer: Malignancy.

Question: Name a potentially life-threatening infection of the auditory external canal which spreads to the soft tissues and to the skull base?

Answer: Malignant otitis externa. Rapid Review Otitis Externa Patient with a history of swimming or moisture exposure Complaining of malodorous discharge and pruritus PE will show pain with palpation of tragus/pinna Most commonly caused by Pseudomonas aeruginosa Treatment is topical antimicrobials with or without steroids Comments: Necrotizing otitis externa - a complication seen in diabetics/immunocompromised

Question: What are the classic viral exanthems?

Answer: Measles (first disease), rubella (third disease), erythema infectiosum (fifth disease), and roseola infantum (sixth disease). Rapid Review Erythema Infectiosum (Fifth Disease) Patient will be a child With a history of URI symptoms 3 - 4 days prior to rash PE will show "slapped cheek" rash Most commonly caused by parvovirus B19 Comments: Aplastic crisis in sickle cell patients

Question: What is the name for the painful, bluish discoloration of the leg associated with extensive DVT burden to the leg?

Answer: Phlegmasia cerulea dolens. Rapid Review Deep Vein Thrombosis (DVT) Patient with a history of smoking, long distance travel, surgery, oral contraceptives use Complaining of unilateral leg edema, leg pain, tenderness and warmth PE will show positive Homan's sign Diagnosis is made by first ultrasound, Gold Standard: venography Most commonly caused by stasis, hypercoagulable state, trauma (Virchow's triad) Treatment is IV heparin and switch to warfarin Warfarin is contraindicated in pregnancy Comments: Risk stratification by Well's criteria

Question: What should be co-administered with isoniazid?

Answer: Pyridoxine (vitamin B6) to prevent peripheral neuropathy. Rapid Review Latent Tuberculosis Positive PPD criteria: 15 mm: no ↑ risk 10 mm: high-risk, homeless, health-care workers, IVDU, foreign born 5 mm: immunosuppressed, recent TB contact, abnormal CXR, steroid use ​Rx: INH for 9 months

Question: Use of an Amsler Grid during vision tests can be helpful for determining progression of which ophthalmologic condition?

Answer: Symptoms of worsening central vision due to macular degeneration can be detected by asking patients to routinely view an Amsler Grid. Rapid Review Macular Degeneration Patient will be older Complaining of bilateral, gradual central field vision loss PE will show Dry macular degeneration (85% of cases): Atrophic changes and yellow retinal deposits (Drusen spots) Wet macular degeneration: Vascular changes Diagnosis is made by Amsler grid Most common cause of blindness in the elderly

Question: Why does a healthy adult with uncomplicated Group A strep pharyngitis generally not require antibiotic treatment?

Answer: The occurrence of the initial episode of acute rheumatic fever is unlikely after 15 years of age.

Question: In what age group are supracondylar fractures common?

Answer: They typically occur in children between 5 and 10 years of age. Rapid Review Radial Head Fractures Type I/II: nonoperative Surgery indications (rule of 3s): >3 mm displacement, >30% of articular surface involvement, >30% angulation exists

Question: True or false: Prophylactic penicillin should be given to all children with sickle cell disease until age five?

Answer: True.

Question: Which nongynecologic sources must be initially considered in any woman with abnormal uterine bleeding?

Answer: Urologic and rectal, as it is difficult for some women to localize bleeding from the vagina, urethra or anus. Rapid Review Dysfunctional Uterine Bleeding (DUB) MCC of abnormal vaginal bleeding in reproductive women Menarche, perimenopause Anovulatory: ↑ Estrogen, ↓ progesterone → endometrial hyperplasia/bleeding Unpredictable bleeding Ovulatory: Predictable bleeding Dx of exclusion Rx: combination OCPs Unstable bleeding: IV estrogen

Question: In which population does Sjogren's most commonly occur?

Answer: Women in their late-forties to mid-fifties. Rapid Review Sjogren's Syndrome Patient will be a woman 40 - 60 years old Complaining of dry eyes (Xerophthalmia) and dry mouth (Xerostomia) Labs will show SSA (anti-Ro) or SSB (anti-La) Diagnosis is made by positive Schirmer test Most commonly caused by autoimmune disorder

A 12-year-old girl presents to the office with anal itching that seems to be worse at night. She has no issues with constipation or any other changes in her bowel habits. On physical exam you see some excoriations around the anus but no tear or palpable hemorrhoid. This first occurred a few days after returning home from a summer camp in northern Michigan, where she was in a cabin with 15 other girls for one month. Which of the following is the most likely diagnosis? Internal hemorrhoid Lyme disease Pinworms Scabies

Correct Answer ( C ) Explanation: Pinworm is the most common helminthic parasite encountered by primary care providers in developed nations. It is acquired by ingesting parasite eggs, and most people remain asymptomatic after being colonized. Pinworm (E. vermicularis) is a quintessential intestinal parasite with no geographic constraints. It is transmissible by close contact with colonized persons. People have had pinworm for thousands of years, and before modern sanitation, colonization by pinworm probably was universal. E. vermicularis has a simple life cycle with a "hand to mouth" existence. The worm is acquired by ingesting parasite eggs. Most often these eggs are on the hands of the host; however, the small eggs also may become airborne, inhaled, and then swallowed. During the night, egg-laden females migrate out of the anal canal and onto the perianal skin. Each female deposits up to 17,000 eggs which mature rapidly, becoming infective within six hours causing extreme pruritus. Infestation typically causes perianal itching and scratching gathers eggs onto the hands, promoting reinfection and transmission to others. Diagnosis is made by the cellophane tape test and all members of the household should be treated. Pinworm infection is readily treated with a single 100-mg dose of mebendazole or a 400-mg dose of albendazole. Internal hemorrhoids (A) can cause anal itching, but are usually palpable on physical exam and are associated with blood in the stool and would not be restricted to night time itching. Lyme disease (B) is uncommon in Michigan and transmitted by a tick. The characteristic bulls-eye (erythema migrans) rash is usually present. Scabies (D) can cause itching in the rectal area but is associated with linear excoriations in regions other than the anus, such as along the waistline, the interdigitus spaces and axillary region.

A 25-year-old woman comes to the office with diffuse pain, multiple tender points and generalized fatigue. A full work up revealed no connective tissue disease but you suspect fibromyalgia. Which of the following can be used for treatment of fibromyalgia? Aerobic exercise High dose opioids Physical therapy Trigger point injections

Correct Answer ( A ) Explanation: Aerobic exercise increases cardiovascular fitness and reduces pain and other fibromyalgia symptoms. Fibromyalgia is characterized by high levels of pain, sleep disturbance, and fatigue combined with a general increase in medical symptoms including problems of memory or thinking and often psychologic distress. Many issues that surround fibromyalgia are not scientific ones—it is widely agreed that pain and suffering are real; instead, the primary issues are often social, political, and financial. Fibromyalgia lies at the end of a continuum of polysymptomatic distress rather than being a discrete disorder. Fibromyalgia may be diagnosed with American College of Rheumatology (ACR) 2010 or 1990 criteria, but clinical care does not require a diagnosis. The ACR 2010 criteria should result in changes in the sex ratio of patients with fibromyalgia because men have higher pain thresholds and are therefore less likely to be diagnosed as having fibromyalgia than women when the 1990 criteria including tender points are used. Although aerobic exercise is beneficial it is felt that formal physical therapy (C) leads to unnecessary use of the health care system and takes the ownership of disease improvement away from the patient. High dose opioids (B) are generally not recommended. Experience has shown that they often do not provide any long-term benefit and may lead to dependence and constipation. There is some evidence for the use of strength training and hypnosis. There is weak evidence for chiropractic, manual, and massage therapy and no evidence of efficacy for tender or trigger point injections (D).

A 16-year-old male football player presents to the emergency room confused and in shock. There is no history of trauma, travel, preceding illness, or animal exposure. The young man appears pale, diaphoretic, and tachycardiac, with a respiratory rate of 24 and a blood pressure of 95/55. Palpation of the slightly swollen right thigh is exquisitely painful, and there is slight erythema over that area. After aggressive fluid administration, an emergency MRI demonstrates a suppurative process involving the fascia of his right thigh. Which of the following is the most appropriate treatment at this time? Administer morphine for pain relief Obtain a CT scan with IV contrast of the right thigh Start intravenous vancomycin and clindamycin Start tetracycline orally

Correct Answer ( C ) Explanation: Start intravenous vancomycin and clindamycin. Streptococcal Toxic Shock Syndrome (Strep TSS), as is true for any disease process manifesting with shock, is a medical emergency. One or more pyrogenic exotoxins secreted by Streptococcus pyogenes (Group A Strep) when it invades host tissues, is responsible for the syndrome. A similar but not identical pyrogenic exotoxin is produced by Staphylococcus aureus and may present with a similar clinical picture. It is, therefore, prudent to start antibiotics that cover both Strep and Staph, such as vancomycin, as soon as possible after treatment for shock is instituted. It has furthermore been demonstrated that treatment of toxic shock syndromes with a beta-lactam antibiotic such as penicillin or ampicillin is clinically inferior to a ribosomally active antibiotic such as clindamycin. This is thought to reflect the ability of clindamycin to turn off toxin production by the invasive bacteria. In addition to antibiotics, these patients often require surgical consultation for possible debridement. The administration of morphine (A) or other drug for pain relief is appropriate and humane but not sufficient because it does not treat the cause of the potentially fatal infection. Obtain a CT scan with IV contrast of the right thigh (B) is not necessary at this time. It is more important to initiate antibiotics. Advanced imaging may be performed after the patient is stabilized. Starting tetracycline (D) or any oral drug is inappropriate in the emergent treatment of a potentially rapidly fatal infection. In addition, it is not a universally effective drug against Group A Strep.

A 67-year-old African-American man established care with you two months ago after being told at a community health fair that his blood pressure was elevated. The blood pressure readings done in your office include 168/94, 158/98 and 162/96. You determine that the patient has primary essential hypertension. Lifestyle measures have already been instituted but you decide that medication is also warranted at this time. Which of the following is the best initial agent for blood pressure control in this patient? Amlodipine Clonidine Metoprolol Prazosin

Correct Answer ( A ) Explanation: Current studies suggest that the best initial choice for treating hypertension in African-American patients without diabetes or chronic kidney disease is either a calcium-channel blocker or a thiazide-type diuretic. When used as monotherapy, either of these agents is more effective at lowering blood pressure than drugs in other classes, though many patients require the addition of a second anti-hypertensive agent for adequate control. Clonidine (B) is a central alpha sympathetic agonist used for the treatment of hypertension. It is associated with several drug-drug interactions and orthostatic hypotension. Although it is used as an anti-hypertensive in some patients, it is not recommended as first-line therapy. Metoprolol (C) is a cardioselective beta-blocker used in the management of hypertension. Beta-blockers have been found to be comparatively less effective agents for the treatment of hypertension in African Americans, and in recent years have fallen out of favor as the best initial option for managing hypertension in other patients groups, as well. Prazosin (D) is one of several alpha-adrenergic receptor blockers which work to lower blood pressure by decreasing peripheral vascular resistance. It is not recommended as a first-line agent.

A 24-year-old woman presents to the Emergency Department after a high-speed motor vehicle collision. Her GCS is 14. On neurological examination, she has intact light touch, but decreased strength and loss of pain and temperature sensation below the T10 level. What is the most likely diagnosis? Anterior cord syndrome Brown-Séquard syndrome Cauda equina syndrome Central cord syndrome

Correct Answer ( A ) Explanation: Injury to the anterior two-thirds of the spinal column results in anterior cord syndrome which is characterized by bilateral loss of motor function and pain and temperature sensation below the level of injury. The dorsal column is usually spared so vibration sense and proprioception are preserved. Anterior cord syndrome occurs in cases of disruption or injury to the anterior spinal artery which can result from protrusion of bone fragments from a traumatic injury to the vertebrae, disc herniation, or following aortic surgery. It can also result from a flexion/compression injury to the cord. Most improvement in function occurs in the first 24 hours. Anterior cord syndrome carries the worst prognosis of all the incomplete spinal cord syndromes, with less than 20% regaining any degree of muscle function. Brown-Séquard syndrome (A) results from hemisection of the spinal cord, often due to penetrating injuries. Patients present with ipsilateral motor, vibration and proprioception loss, and contralateral pain and temperature loss. Cauda equina syndrome (C) is typically seen in cases of midline disk herniation at the L4-5 level that results in dysfunction of nerve roots, not the spinal cord itself. Patients present with urinary retention with overflow incontinence, fecal incontinence, distal motor weakness, and sensory loss in a saddle distribution. Central cord syndrome (D) is characterized by bilateral motor paresis with the upper extremities affected to a greater degree than the lower extremities. Sensory loss is variable.

A 7-year-old boy presents to the emergency department with cough and chest pain. The cough has been present for several days, but the chest pain has been present for only one day. His vital signs are notable for a temperature of 38.8°C, heart rate of 150, blood pressure of 78/49, respiratory rate of 50, and oxygen saturation of 94%. He is ill appearing, has coarse breath sounds on lung auscultation, a systolic murmur and a gallop on his cardiac exam, and his liver is palpable 4 centimeters below the right costal margin. His pulses are palpable and capillary refill is 5 seconds. Which of the following is the most likely complication? Aortic valve insufficiency Constrictive pericarditis Dilated cardiomyopathy Left ventricular outflow tract obstruction

Correct Answer ( C ) Explanation: This patient has acute systolic dysfunction in the setting of an upper respiratory infection, which is most likely secondary to myocarditis. Myocarditis can be caused by infections, connective tissues disease, genetic diseases, toxins, and granulomatous diseases. The most common viral etiologies are enterovirus, adenovirus, parvovirus, EBV, and CMV. Myocardial damage may occur from inflammation and direct damage to the myocardium. This results in ventricular enlargement, poor systolic function, and dilated cardiomyopathy. The systolic dysfunction may result in congestive heart failure, arrhythmias, and even death. Patients typically present with chest pain, fever, and fatigue, and in fulminant disease they may have respiratory distress, tachycardia, hypotension, a gallop rhythm, and a murmur. Rash and end-organ involvement (i.e. hepatitis) may be seen. Dilated cardiomyopathy is the most common type of cardiomyopathy, and is characterized by left ventricular systolic dysfunction and left ventricular dilation. In pediatrics, it is usually caused by genetic mutations, myocarditis, or Kawasaki disease. Aortic valve insufficiency (A) is a possible complication of rheumatic fever. Rheumatic fever is characterized by carditis, migratory polyarthritis, erythema marginatum, subcutaneous nodules, and chorea. Constrictive pericarditis (B) is associated with viral infections and chest pain. However, the clinical findings seen in this disorder would be muffled heart sounds, a narrow pulse pressure, jugular venous distension, and a pericardial friction rub. Left ventricular outflow tract obstruction (D) is a finding in hypertrophic cardiomyopathy. Hypertrophic cardiomyopathy would not be expected to develop after myocarditis.

An 83-year-old woman with a history of constipation presents to the ED complaining of a rectal mass. On exam, you note the mass seen in the image above. What is the most appropriate next step in management? Attempt manual reduction Consultation to a colorectal surgeon Immediately begin broad-spectrum antibiotics Inject local anesthesia and perform an excisional thrombectomy

Correct Answer ( A ) Explanation: Rectal prolapse is a disease of the very young and the very old. In adults, it is most commonly seen in older women and is most often associated with excessive straining while defecating, coughing, or sneezing. This is due to laxity of attachment structures and is often accompanied by prolapse of the bladder (cystocele) and uterus. Often, bloody mucous discharge will be seen along with fecal incontinence and dull pain. Manual reduction is usually sufficient for most cases and should be attempted in the ED. In some cases, the rectal tissue becomes edematous and at risk for vascular compromise. These cases require urgent surgical consultation (B). Antibiotics (C) are not necessary in adult rectal prolapse. Pediatric rectal prolapse is more worrisome and is associated with malnutrition, parasitic infection, and cystic fibrosis. An excisional thrombectomy (D) is reserved for thrombosed external hemorrhoids, not rectal prolapse.

Which of the following is used to diagnose and monitor scoliosis? Cobb angle Hoffman reflex Modified Schober test Pelvic tilt

Correct Answer ( A ) Explanation: Scoliosis is lateral curvature of the thoracic or lumbar spine > 10 degrees. It is usually idiopathic, but can be secondary to neuromuscular disease (cerebral palsy, spinal muscular atrophy, myelomeningocele), vertebral disease (tumor, infection), neurofibromatosis, or Marfan syndrome. Symptoms usually develop in early adolescence. Females are more likely to have progressive disease requiring treatment. Adult onset scoliosis is commonly caused by spinal degenerative disk disease, facet spondylosis and spondylolisthesis. On exam, use a forward bending test to assess for vertebral and rib rotation. The diagnosis is made radiographically with weight-bearing PA and lateral full-length spinal views to measure the Cobb angle. The Cobb angle is used to quantify the magnitude of spinal deformities. To measure the Cobb angle, first decide which vertebrae are the end-vertebrae of the curve deformity (vertebrae at the upper and lower limits of the curve) and then Cobb angle formed by the intersection of two lines: one parallel to the endplate of the superior end vertebra and the other parallel to the endplate of the inferior end vertebra. The angle is plotted and scoliosis is defined as a lateral spinal curvature with a Cobb angle of 10° or more. The Hoffman reflex (B) involves assessing for thumb or second digit flexion when the nail of the third digit is flicked. A positive response suggests upper motor neuron disease such as cervical spinal cord injury. The modified Schober test (C) is used to quantify lumbar spine flexion and extension in the sagittal plane. It is not used to monitor worsening coronal spine curve as in progressive scoliosis. Pelvic tilt (D) refers to the height difference between the iliac crests. It is primarily used to evaluate leg-length discrepancy.

Which of the following disease entities can be treated with the Epley maneuver? Benign paroxysmal positional vertigo Meniere's disease Vertebrobasilar insufficiency Vestibular neuritis

Correct Answer ( A ) Explanation: The Epley maneuver may be curative in patients with benign paroxysmal positional vertigo (BPPV). Vertigo is defined as the sensation or illusion of movement, often spinning, when the patient is standing still. There are numerous causes of vertigo. For matters of diagnosis and management, they are often divided into peripheral causes and central causes. Peripheral causes involve pathophysiologic changes outside of the central nervous system and are typically benign (with the exception of acoustic neuroma). Central vertigo, on the other hand, is generally caused by a dangerous etiology affecting the central nervous system (vertebrobasilar insufficiency, cerebellar hemorrhage and occlusion of the posterior inferior cerebellar artery). BPPV typically involves rapid onset of vertigo with nausea and vomiting that is elicited with moving the head to a certain position and relieved by moving the head away from that position. It is typically caused by the presence of an otolith in one of the semicircular canals in the inner ear. A Dix-Hallpike test can be performed to aid in diagnosis and an Epley maneuver can be used to dislodge the otolith from the semicircular canal leading to a resolution of symptoms. Meniere's disease (B) is defined as longer episodes of vertigo with associated tinnitus and is not caused by an otolith. Verebrobasilar insufficiency (C) is a central cause of vertigo and should be treated as a vascular ischemic episode. Vestibular neuritis (D) is caused by inflammation and does not respond to the Epley manuver.

Which of the following medications commonly used to treat hyperemesis gravidarum is considered Category B? Metoclopramide Prochlorperazine Promethazine Trimethobenzamide

Correct Answer ( A ) Explanation: The FDA has developed a classification system to categorize mediations based on potential teratogenic effects. Medications are assigned to one of the following categories based on human and animal data. Metoclopramide is a Category B antiemetic. It is a centrally acting dopaminergic anatagonist that is used in the treatment of nausea and vomiting. Prochlorperazine (B), promethazine (C), and trimethobenzamide (D) are all Category C agents. Prochlorperazine and promethazine are also dopamine antagonists. The mechanism of action for trimethobenzamide is not well understood. Ondansetron (trade name Zofran) another commonly prescribed antiemetic is also Category B.

A 3-year-old girl with a past medical history of constipation is brought to the ED for evaluation of a limp and left knee pain. Her physical exam is unremarkable. You obtain the knee radiograph seen above. What do you expect to see on her peripheral blood smear? Basophilic stippling Heinz bodies Hypersegmented neutrophils Schistocytes

Correct Answer ( A ) Explanation: This patient has lead lines on her knee radiograph, a classic finding of lead poisoning. These hyperdense lines occur at the metaphyses and are most commonly seen in young children on knee and wrist radiographs. Similar lines may also be seen in other heavy metal poisoning such as bismuth and arsenic, and in healing rickets or scurvy. Lead poisoning causes a microcytic, hypochromic anemia. On peripheral blood smear, the pathognomonic finding is basophilic stippling. Basophilic stippling represents clumps of degraded RNA that have not been eliminated due to lead-induced nucleotidase inhibition. This child's joint pain and constipation are likely related to lead poisoning. Heinz bodies (B) are inclusions within red blood cells made of denatured hemoglobin. They are seen in glucose-6-phosphate deficiency, liver disease, and alpha-thalassemia. Hypersegmented neutrophils (C) are seen in multiple forms of megaloblastic anemia such as that caused by alcohol abuse, folate deficiency, and vitamin B12 deficiency. Schistocytes (D) are fragmented red blood cells caused by shearing with fibrin strands in microarterioles. They are seen on smears in patients with artificial heart valves and in numerous disease processes, including hemolytic uremic syndrome, thrombotic thrombocytopenic purpura, disseminated intravascular coagulation, and congestive heart failure.

A 43-year-old man with a history of alcoholism presents with generalized weakness. Physical examination reveals pale conjunctiva. The patient's hematocrit is 25% and his mean corpuscular volume is 110 fL. Which of the following treatments is most likely indicated? Folic acid supplementation Lead chelation therapy Splenectomy Thiamine supplementation

Correct Answer ( A ) Explanation: This patient presents with megaloblastic anemia likely due to folic acid deficiency secondary to alcohol abuse. Megaloblastic anemia is defined as anemia in the presence of macrocytic red blood cells. It is an important cause of reversible anemia. Megaloblastic anemia is the manifestation of defective DNA synthesis in response to a lack of vitamin B12 and folic acid. Rapid hematopoietic cell turnover leads to ineffective erythropoiesis and pancytopenia. Differentiating folic acid deficiency from vitamin B12 deficiency requires serum testing. Folic acid deficiency is common in those with inadequate dietary intake from chronic alcohol use and increased use, as in pregnancy. B12 deficiency, on the other hand, usually results from chronic malabsorption. Supplementation with 1 mg of folic acid every day can lead to resolution of this disorder. Lead chelation therapy (B) is indicated for lead toxicity causing a microcytic anemia, not macrocytic. Basophilic stippling is commonly seen in lead toxicity as well. Splenectomy (C) can be therapeutic in the management of Thalassemias. Thiamine (D) (vitamin B1) is often deficient in chronic alcoholics as well but does not cause anemia. Thiamine is required in glucose metabolism and can result in Beriberi or Wernicke-Korsakoff syndrome.

What is the most common infectious etiology of conjunctivitis in adults? Adenovirus Enterovirus Staphylococcus aureus Streptococcus pneumoniae

Correct Answer ( A ) Explanation: Viral conjunctivitis is the most common type of conjunctivitis in adults and is most often caused by adenovirus. Patients present with symptoms of redness, watery or mucoserous discharge, and irritation. It is typically bilateral (but may initially affect only one eye) and there is often a preceding or concomitant upper respiratory illness. Physical examination findings include preauricular lymphadenopathy, diffuse conjunctival injection and enlarged follicles on the inferior palpebral conjunctiva. Visual acuity, examination of the cornea and anterior chamber, and intraocular pressures are normal. Treatment consists of supportive care with cool compresses. Some patients may get relief from an over-the-counter topical antihistamine or ocular decongestant but this treats only the symptoms and not the cause. Topical antibiotics are reserved for those patients in whom the etiology is unclear or if there is concern for a superimposed bacterial infection. Enterovirus (B) may also cause viral conjunctivitis, but is less frequently implicated than adenovirus. Staphylococcus aureus (C) and Streptococcus pneumoniae (D) are common causes of bacterial conjunctivitis, but neither are responsible for the majority of cases overall.

Which of the following characteristics helps to differentiate delirium from dementia? Attention is spared in delirium Delirium has an acute onset Delirium has no change in level of consciousness PET scanning is abnormal in delirium

Correct Answer ( B ) Explanation: An important distinction between delirium and dementia is the time of onset. Delirium is characterized by its rapid onset and fluctuating course. It leads to acute changes in mental status, marked by inattention, poor concentration, and fluctuating levels of consciousness. It commonly occurs in hospitalized elderly patients. Many disorders can have similar clinical features of delirium and patients with delirium are often mistakenly diagnosed with dementia or depression. Delirium, dementia, and depression can coexist within the same patient. When an acute change in mental status occurs, however, the diagnosis of delirium should be presumed until proven otherwise. Treatment involves identifying and treating the underlying causes. Clinicians should minimize the use of physical restraints as much as possible and encourage consistent presence of family or caregivers familiar to the patient. Inattention (A) is a distinguishing feature between delirium and dementia. Attention is spared in early and moderate dementia, whereas it is always impaired in delirium. Altered levels of consciousness (C) should not be present in mild or moderate dementia or depression, however it is seen in delirium. PET scanning (D) cannot differentiate acute delirium from dementia.

A 14-year-old girl sprained her ankle. She rates her pain 5/10. On examination, she has moderate tenderness and swelling with decreased range of motion secondary to pain. Although quite painful, she is able to ambulate. What is the grade of this ankle sprain? Grade I Grade II Grade III Grade IV

Correct Answer ( B ) Explanation: Ankle sprains are caused by partial or complete tearing of one or more ligaments that support the ankle joint. Injuries are most often due to an inversion mechanism, causing injury to the lateral ligaments. The anterior talofibular ligament (ATFL) is the first ligament to be injured, followed by the calcaneofibular ligament (CFL), and finally, in the most severe lateral sprains, the posterior talofibular ligament (PTFL). A syndesmotic ankle sprain (high ankle sprain) involves the ligaments that connect the tibia and fibula. Ankle sprains are graded I, II or III based on exam findings and functional loss. The patient in the above scenario has a Grade II ankle sprain (incomplete tear) characterized by moderate pain, swelling, tenderness and ecchymosis with mild to moderate joint instability and some loss of range and function. Typically these are painful with weight bearing. Grade I (A) is characterized by mild stretching and microscopic tears of the ligament. Mild swelling and tenderness is noted with no joint instability. The patient is able to bear weight and ambulate with minimal pain. Due to their benign nature, these injuries are not frequently seen in the healthcare setting. Grade III (C) is characterized by a complete tear of the ligament. Severe pain, swelling, tenderness and ecchymosis is noted with significant joint instability. There is significant loss of range and function and the patient cannot bear weight or ambulate. There is no Grade IV (D) ankle sprain.

An 83-year-old is being evaluated in the emergency department after an episode of syncope. The woman was preparing dinner when she felt her heart start to race. The next thing she remembers is waking up on the floor. She experienced a similar episode about three weeks ago. She has never had anything like this before. Her past medical history is remarkable for hypertension, hyperlipidemia and hypothyroidism. Her medications include lisinopril, atorvastatin and levothyroxine. On physical exam her blood pressure is 142/83, heart rate 76/min, and respiration rate 13/min. Cardiac auscultation reveals no murmur. The remainder of her physical exam is normal. Electrocardiogram reveals normal sinus rhythm with left axis deviation. No cardiac rhythm abnormalities are detected. What is the most likely etiology of this patient's syncope? Aortic stenosis Cardiac dysrhythmia Orthostatic hypotension Vasovagal

Correct Answer ( B ) Explanation: Cardiac dysrhythmia is the most likely cause of this woman's syncope. Cardiac dysrhythmias are a common cause of syncope in the elderly population. It is characterized by a brief or absent prodrome and palpitations immediately preceding the event. Several episodes over a short period of time in someone with no history of syncope suggest a dysrhythmia. Given this patient's short prodrome, palpitations and history of a previous similar event makes a cardiac dysrhythmia the most likely etiology. Aortic stenosis (A) is unlikely the cause of her syncope. Aortic stenosis is associated with a crescendo-decrescendo systolic ejection murmur. Syncope related to aortic stenosis typically occurs during exertion and is associated with very severe disease. This patient's syncopal episode occurred while stationary. Additionally, she has no systemic symptoms of aortic stenosis. Vasovagal (D) is the most common cause of syncope in the general population. It is usually triggered by provoking factors such a blood draw or an intense emotion. Prodromal symptoms include feeling warm, sweating, nausea, and pallor. This woman does not report any of these symptoms. Orthostatic hypotension (C) causes syncope upon assuming an upright position from supine or sitting. It is often caused by hypovolemia, medications or autonomic nervous system disorders. This woman was standing while preparing dinner making orthostatic hypotension unlikely.

Which of the following is a class IC anti-arrhythmic? Amiodarone Flecainide Lidocaine Procainamide

Correct Answer ( B ) Explanation: Flecainide is a class IC anti-arrhythmic drug. All class I anti-arrhythmics exert their action at the fast sodium channels. The subsets of class I relate to how the drugs affect depolarization, repolarization and conduction. The class IC drugs slow depolarization and conduction. Flecainide is most commonly used in the treatment of supraventricular tachycardias as well as ventricular tachycardia not related to acute ischemia. Procainamide (D) is a class IA drug that is the most common drug in this class used for treatment of supraventricular tachycardias and ventricular tachycardias in the emergency setting. Class IA agents slow conduction through the atria, AV nodes and His-Purkinje systems. Additionally, conduction through accessory pathways is slowed. Infusion of procainamide may lead to hypotension and QRS widening. Lidocaine (C) is the only Class IB agent used in emergent treatment of dysrhythmias particularly those resulting from increased automaticity like ventricular tachycardia. These agents work by slowing conduction and depolarization and shortening repolarization. Amiodarone (A) is a class III drug, although it also has characteristics of Class IA, II and IV agents. The class III drugs prolong the refractory period by blocking potassium channels.

A 18-year-old woman presents with a diffuse papulosquamous rash. The rash began 1-month prior when she noticed a large patch on her neck that was followed by the diffuse papulosquamous rash. You inform the patient that the rash will last 5-8 weeks and prescribe her cetirizine. Which of the following is the most likely diagnosis? Contact dermatitis Pityriasis rosea Scabies Tinea versicolor

Correct Answer ( B ) Explanation: Pityriasis rosea is a common acute eruption usually affecting children and young adults; the cause is unknown. It is characterized by the formation of an initial herald patch, followed by the development of a diffuse papulosquamous rash. Pityriasis rosea is difficult to identify until the appearance of characteristic, smaller, secondary lesions that follow Langer's lines. The rash of pityriasis rosea typically lasts 8 to 12 weeks, with complete resolution in most patients. An important goal of treatment is to control pruritus, which may be severe; zinc oxide, calamine lotion, topical steroids, and oral antihistamines are usually helpful. Systemic steroids are generally not recommended. Patients should be reassured about the self-limited nature of pityriasis rosea. Persistence of the rash or pruritus beyond 12 weeks should prompt reconsideration of the original diagnosis, consideration of biopsy to confirm the diagnosis, and questioning the patient again about use of medications that may cause a rash similar to that of pityriasis rosea. Findings of contact dermatitis (A) can include erythema, vesicles, bullae, exudation, and crusting from breaking of blisters, swelling, and scaling. Taking a careful history and patch testing are often the key to diagnosis. In scabies (C) patients present with a pruritic rash that is often worse at night. Skin findings include papules, nodules, burrows, and vesiculopustules. The distribution includes the interdigital spaces, wrists, ankles, waist, groin, and axillae. Pruritic nodules around the axillae, umbilicus, or on the penis and scrotum are highly suggestive of scabies. Tinea versicolor (D) presents with hypopigmented, pink/brown macules and patches on the trunk with fine scale. Versicolor means varied colors, and this tinea tends to be white, pink, and brown. Tinea versicolor is found on the back, chest, abdomen, and upper arms, often in a cape-like distribution.

Which of the following is the most common cause of a pleural effusion in developed countries? Bacterial pneumonia Heart failure Malignancy Tuberculosis

Correct Answer ( B ) Explanation: Pleural effusions occur when fluid accumulates between the parietal and visceral pleurae. In a healthy patient, fluid is continually produced by the parietal pleura which reduces friction allowing for smooth lung expansion. If there is overproduction or decreased reabsorption, fluid can accumulate in the potential space. Depending on the volume present, pleural effusions can be clinically silent or cause symptoms, such as dyspnea or chest pain. On physical exam, there will be decreased breath sounds and dullness to percussion. Pleural effusions can be seen on upright chest X-ray when there is more than 150-200 cc of fluid present. Smaller effusions may only be seen on left lateral decubitus films or CT scan. Pleural effusions can be classified as transudates or exudates based on Light criteria. Fluid is an exudate if one or more of the following are present: (1) pleural fluid/serum protein ratio > 0.5, (2) pleural fluid/serum LDH > 0.6, (3) pleural fluid LDH is > 2/3 of the upper limit for serum LDH. Common causes of transudates include congestive heart failure, cirrhosis, and nephrotic syndrome. Common causes of exudates include malignancy, infection, and pulmonary embolism. Overall, the most common cause of a pleural effusion in the Western world is heart failure. Bacterial pneumonia (A) and malignancy (C) are causes of an exudative effusions and the third and second most common cause of effusions overall. Tuberculosis (D) is the leading cause of pleural effusions outside the Western world.

Which of the following is the first line treatment for latent TB infection? Doxycycline Isoniazid Lamivudine/Zidovudine Penicillin

Correct Answer ( B ) Explanation: Standard therapy for latent TB infection (LTBI) is isoniazid (INH). In the United States, latent tuberculosis infection is the most prevalent form of tuberculosis. LTBI is the term given to patients with a positive purified protein derivative (PPD) skin test without evidence of active TB. PPD has been used for more than 100 years and relies on delayed-type hypersensitivity (DTH) to M. tuberculosis cellular proteins. Because PPD relies on DTH, any factor that reduces the DTH affects the host response to PPD. The most common clinical example is use of corticosteroids, which blunt the DTH response and can complicate PPD interpretation. Therefore, PPD testing should not be performed while a patient is taking corticosteroids. Also, TB testing should be targeted to those with higher risk of infection and should not routinely be done in those with low risk. Patients at increased risk for progression to active TB include those who have been recently infected (recent PPD converters); patients who are HIV seropositive; patients who have silicosis, diabetes, or chronic renal failure (including those receiving hemodialysis); solid-organ transplant recipients; patients with gastrectomy or jejunoileal bypass or head and neck cancer; injection drug users; patients with chest radiograph evidence of prior TB; and patients who weigh at least 5% less than ideal body weight. Patients taking chronic corticosteroid therapy and those who are to receive tumor necrosis factor alpha (TNF-alpha) blockers (e.g., infliximab) are also at risk. Lamivudine/Zidovudine (C) is used for post-exposure prophylaxis for prevention of HIV infection. Doxycycline (A) and penicillin (D) is the treatment in a variety of bacterial infections, but not latent tuberculosis.

A 57-year-old man with hypertension presents complaining of a 6-hour history of a "floater" in the right eye. He states that he has had increasing difficulty reading as the cloudy area blocks his visual field. Additionally, he complains of decreased vision in his right eye. Which of the following represents the appropriate management? Administer timolol drops and emergently consult ophthalmology Emergent ophthalmology consultation Prescribe topical antibiotics and refer to ophthalmology Refer patient to ophthalmology for further assessment

Correct Answer ( B ) Explanation: This patient is suffering from a posterior vitreous detachment and should have an emergent consultation with ophthalmology. Posterior vitreous detachment involves separation of the posterior vitreous from the retina resulting from vitreous degeneration and shrinkage. It can be accompanied by a retinal detachment as well. Floaters (the sensation of dark spots moving in the visual field) are a common presentation of posterior vitreous detachment. Floaters that are stable or chronic may be referred to ophthalmology on a non-emergent basis (D) but acute floaters should be emergently assessed as they may rapidly progress. Any subjective or objective change in vision or visualization of vitreous pigment or hemorrhage on slit-lamp evaluation suggests an acute posterior vitreous detachment that should be emergently seen by an ophthalmologist. Acute angle closure glaucoma can present with a change in vision but is usually accompanied by ocular pain. This disorder should be treated immediately with a topical beta-blocker (A) and have an emergent ophthalmology evaluation. Corneal abrasions over the visual axis can also cause a change in vision but are almost always painful. Treatment with topical antibiotics (C) and ophthalmology referral is appropriate.

An 18-month-old boy is brought in by his parents for shortness of breath. The parents woke to him coughing a low-pitched cough. They also noted other noises when he was breathing in that resolved upon walking outside. The patient is frequently coughing but has no abnormal sounds on auscultation of the neck or lungs. What is the most appropriate treatment? Albuterol Dexamethasone Racemic epinephrine Ribavirin

Correct Answer ( B ) Explanation: This patient is suffering from croup, or laryngotracheobronchitis. Croup is a viral infection of the upper airway, most commonly caused by parainfluenza virus. It is the most common cause of upper airway distress and obstruction in children. The peak incidence of croup is 2 years and the classic age range of children affected is 6 months to 6 years. Most cases occur in late fall, winter and early spring. Patients have a characteristic barking cough that sounds like a seal. Symptoms occur because of edema and exudate of the subglottic space. Croup is a clinical diagnosis based on the presence of the barking cough, hoarse voice and sometimes stridor. Preceding the development of these symptoms, children may have had a prodrome of fever and URI symptoms. The administration of dexamethasone is the mainstay of therapy and has been shown to decrease the duration of symptoms, decrease return visits to the ED, decrease length of stay in the ED and decrease the need for epinephrine. In patients with stridor, the administration of racemic epinephrine leads to rapid improvement of symptoms in most cases. Patients who have received racemic epinephrine need a period of observation after treatment for 1-2 hours to ensure that stridor and retractions do not recur. On X-ray, the steeple sign is present due to a narrowing of the upper trachea from the infection. Albuterol (A) is a beta-agonist used in the treatment of bronchoconstriction. Beta-agonists have no benefit in the treatment of patients with croup, as this is a disease of the upper airway. Racemic epinephrine (C) is typically reserved for patients with significant or resting stridor or signs of respiratory distress with retractions. After the administration of nebulized racemic epinephrine, patients must be observed for recurrent symptoms. Ribavirin (D) is an anti-viral medication sometimes used in the management of hepatitis C, but also is used off-label in the treatment of some respiratory syncytial virus (RSV) infections.

Which of the following best characterizes diabetes insipidus? High urine osmolality, high serum osmolality High urine osmolality, low serum osmolality Low urine osmolality, high serum osmolality Low urine osmolality, low serum osmolality

Correct Answer ( C ) Explanation: Diabetes insipidus (DI) causes polyuria and in inability to concentrate urine because of a deficiency of antidiuretic hormone (vasopressin). ADH acts directly on the renal collecting system and distal tubules to reabsorb water from the urine back into the serum, hence the term "antidiuretic." There are several forms of DI, but the most common types are central and nephrogenic. Central (neurogenic) DI is due to a decrease in the release of ADH. Causes include neurosurgery, trauma, tumors, ischemia, infiltrative diseases, or idiopathic. Head trauma and brain injury can lead to DI. Nephrogenic DI is due to resistance to ADH at the level of the kidneys. DI is characterized by inappropriately dilute urine (low urine osmolality) in the setting of increased serum osmolality. Polyuria and polydipsia are also seen. The treatment for central DI is to treat the underlying lesion and administer intranasal DDAVP (desmopressin synthetic ADH). Nephrogenic DI is treated by treating the underlying condition. Thiazide diuretics and amiloride may help. The urine osmolality is low, not high (A and B) in DI. The serum osmolality is high, not low (B and D) in DI. High urine osmolality and low serum osmolality is consistent with syndrome of inappropriate ADH release (SIADH).

A 52-year-old man comes to the clinic complaining of difficulty swallowing, upper abdominal pain, and heartburn. He says he has a difficult time swallowing solid foods and the food frequently gets "stuck" in his throat. He has a history of food allergies, asthma, and atopic dermatitis. Laboratory studies show markedly elevated serum IgE levels. Upper endoscopy shows stacked circular rings. Which of the following is the most likely diagnosis? Achalasia Esophageal stricture Esophagitis Gastroesophageal reflux disease (GERD)

Correct Answer ( C ) Explanation: Dysphagia to solids, upper abdominal pain, GERD-like symptoms (eg, heartburn), food impaction, and a history of allergies suggests eosinophilic esophagitis (EE). EE is an allergic inflammatory condition of the esophagus that primarily involves eosinophils. EE is strongly associated with allergic diseases (eg, food allergies, asthma, atopic dermatitis). Laboratory studies typically show elevated levels of serum IgE. Endoscopic findings typically show stacked circular rings that can be transient or fixed along with white nodules with granularity. Treatment usually consists of dietary modification (avoidance of food allergens), medical therapy (treatment with corticosteroids and other anti-inflammatory agents), and mechanical dilatation of the esophagus. Achalasia (A) usually results from a defect in the lower esophageal sphincter (LES). These patients typically present with dysphagia to solids AND liquids and regurgitation of undigested food. An esophageal stricture (B) is a benign narrowing of the esophagus that leads to dysphagia. These patients typically present with heartburn, coughing, dyspnea, frequent burping, odynophagia, and hematemesis. Although this patient most likely has underlying gastroesophageal reflux disease (D), the constellation of findings supports a diagnosis of esophagitis.

A 44-year-old woman presents to the Emergency Department with palpitations following the death of her mother 2 days prior. Her ECG is consistent with normal sinus rhythm with a QTc interval of 520 msec. Her heart rate is 86 and blood pressure is 117/82. Her medications include lisinopril and metformin. She took lorazepam yesterday for anxiety related to the loss of her mother. What is the most appropriate therapy? Administer adenosine Discontinue lorazepam Initiate metoprolol Perform vasovagal maneuvers

Correct Answer ( C ) Explanation: Long QT syndrome is defined by a QT interval of more than 460 msec for women and more than 440 msec for men on ECG. It is more commonly caused by medications than by familial prolongation. Physical and emotional stress are common triggers. Manifestations include palpitations, syncope and cardiac arrest. Beta-blockade with medications such as metoprolol may be appropriate in patients who are hemodynamically stable. Cardiology consult or referral is recommended because implantation of a cardiac defibrillator can be beneficial for those at risk of sudden cardiac death. Adenosine (A) is recommended in patients with palpitations due to supraventricular tachycardia. Lorazepam (B), however, is not associated with QT prolongation. Vasovagal maneuvers (D) are recommended in patients with palpitations due to supraventricular tachycardia.

An obese 37-year-old woman is in the emergency room for right-sided abdominal pain and excessive flatulence. This episode has persisted for several hours. On physical exam you palpate her right upper quadrant while she takes a deep breath. The patient experiences pain and has a transient pause in inspiration. This physical exam finding is associated with which of the following signs? Levine sign McBurney's point tenderness Murphy's sign Psoas sign

Correct Answer ( C ) Explanation: Murphy's sign is positive in patients with acute cholecystitis. During inspiration the lungs expand and the diaphragm moves downward. The downward movement pushes the inflamed gallbladder against the examiner's hand which elicits pain. The gallbladder is found in the right upper quadrant just beneath the liver. Acute cholecystitis occurs when a gallstone blocks the cystic duct which prevents the outflow of bile from the gallbladder. This commonly causes pain after eating since bile is used to emulsify fats ingested from a meal. McBurney point tenderness (B) is found in the right lower quadrant between the umbilicus and the superior iliac spine. This is the general location of where the appendix is connected to the cecum and helps diagnose acute appendicitis. Deep palpation of this area irritates the inflamed appendix eliciting pain. The psoas sign (D) is positive when a patient experiences increased abdominal pain when the right leg is extended at the hip while the patient lies on the left side. This maneuver is used to help diagnose appendicitis. The Levine sign (A) is positive when a patient is holding a clenched fist over their chest to describe dull, pressing chest pain consistent with the discomfort of angina pectoris.

Which of the following is associated with myxedema coma? Hyperglycemia Hypernatremia Hypotension Tachycardia

Correct Answer ( C ) Explanation: Myxedema coma is a life-threatening condition of hypothyroidism with a classic presentation of bradycardia, hypothermia, hypotension (D), and altered mental status. It is often seen in patients with undiagnosed or untreated hypothyroidism with a superimposed stressful precipitating event such as infection, surgery, or trauma. On physical exam, patients will often have scant body hair, puffy eyes, and nonpitting edema. Hypoxemia and hypercapnia are common. Treatment includes supportive therapy, such as fluids and glucose, hydrocortisone and IV levothyroxine. Given myxedema coma is clinically difficult to distinguish from sepsis, antibiotics are often administered as well. On laboratory testing, myxedema coma often will show hyponatremia (B) hypocalcemia, and hypoglycemia (A). It is also associated with bradycardia, not tachycardia (D).

Which of the following is necessary to confirm the diagnosis of scoliosis? Adam's forward bend test Magnetic resonance imaging Radiography Scoliometer

Correct Answer ( C ) Explanation: Scoliosis is a lateral curvature of the spine. Idiopathic scoliosis is the most common type of scoliosis and within this category, the most common type is adolescent idiopathic scoliosis (AIS). AIS is defined as a Cobb angle greater than or equal to 10 degrees, age of onset 10 years or greater and no identifiable etiology. The Cobb angle is determined on radiography and is the measurement used to monitor the progression of scoliosis. Risk factors for the development of AIS include age less than 12 years, onset prior to menarche, curves with a Cobb angle greater than or equal to 20 degrees, female sex, and double or thoracic curves. The curvature of scoliosis can progress during periods of rapid growth, such as adolescence. This can cause significant deformity and other clinical manifestations such as cardiopulmonary compromise, therefore it is important to identify and monitor patients with this condition. A number of screening measures are used to determine the likelihood of scoliosis, but radiography is needed for the diagnosis of scoliosis as determined by the Cobb angle. Physical examination for scoliosis includes the Adams forward bend test (A). The patient is observed from the back while bending forward at the waist until the spine is parallel to the ground. Lumbar or thoracic prominence on one side indicates possible scoliosis and further workup should be initiated. Magnetic resonance imaging (B) is indicated when plain radiography is suggestive of an intraspinal pathology, such as a tumor or infection. Scoliosis can be diagnosed on X-ray and MRI is unnecessary. A scoliometer (D) is a tool used for scoliosis screening and can quantify the amount of trunk rotation. Accurate use of the scoliometer is based on operator experience.

Which of the following contributes to immune deficiency in patients with sickle cell disease? Bone marrow hyperplasia Delayed puberty Functional asplenia Vaso-occlusive crises

Correct Answer ( C ) Explanation: Sickle cell anemia is an autosomal recessive genetic disorder that causes a mutated form of hemoglobin S. This mutation leads to chronic vaso-occlusive crises in affected individuals and causes a number of other health problems. It is most commonly found in persons of African ancestry. Generally diagnosed in early childhood, patients with sickle cell disease experience clinical manifestations that include acute and chronic pain, anemia, splenic sequestration, infection, and involvement of multiple organ systems. During the first year of life, the spleen enlarges due to the presence of a large number of sickled cells, also known as sequestration crisis. This sickling of the red blood cells within the spleen leads to splenic infarction. Eventually, the spleen becomes fibrotic and shrinks, resulting in reduced splenic function or functional asplenia. The lack of splenic function causes patients with sickle cell disease to become immunocompromised. Prevention of infection is an important aspect in the treatment of sickle cell disease, and these patients require a specialized vaccine regimen best managed by a specialist in the area of sickle cell disease. Bone marrow hyperplasia (A) is a process found in patients with sickle cell disease that leads to bone and joint destruction. Delayed puberty (B) is seen in children with sickle cell disease. The pathogenesis of this phenomenon is unknown. Vaso-occulsive crises (D), previously referred to as sickle cell crises, are the hallmark clinical manifestation of sickle cell disease.

A 52-year-old woman with a 10-year history of rheumatoid arthritis presents to the ED with blurred vision. For the past 4 weeks, she has noticed significant eye dryness, gritty sensation and burning. The only thing that has helped with this discomfort is excessive rubbing of eyeball. You readily appreciate a swollen, erythematous left eye with purulent drainage and injected conjunctiva. As you are filling out paperwork for ophthalmologic antibiotics and ophthalmology referral, she asks what she can do about her annoying dry mouth. Which of the following medications do you most likely prescribe? Cyclosporine Nortriptyline Pilocarpine Ranitidine

Correct Answer ( C ) Explanation: Sjogren's syndrome is an autoimmune inflammatory condition that mainly affects the tear and salivary glands. It can occur alone, or in conjunction with another rheumatologic condition, such as rheumatoid arthritis or systemic lupus erythematosus. The common symptoms, called sicca symptoms, include dryness of the eyes, mouth (xerostomia), throat and skin. Complaints include eye burning and gritty sensation, mouth dryness with difficulty swallowing and acid reflux, and nasal, throat and skin dryness. Other symptoms include salivary and tear gland tenderness and edema. Overall, the dryness can lead to eye infections, corneal damage, dental decay and gingivitis and oral infections like thrush. In addition, patient's with Sjogren's can have arthralgias, arthritis, myalgias, rash and Raynaud's phenomenon. Treatment begins with gum chewing, artificial tears and saliva, drinking more water, humidifiers, nasal saline spray and regular dental and ophthalmology care. Prescription medications, such as pilocarpine or cevimuline, can be used to simulate the production of saliva. Pilocarpine's muscarinic-receptor-agonist action offers xerostomia patients a parasympathomimetic saliva-producing effect. Cyclosporine (A) can increase tear production, and is mainly used in treating the dry eye or swollen tear gland pain complaints of Sjogren's syndrome. It is not used to treat the dry mouth complaints. Nortriptyline (B), and other tricyclic antidepressants, are notoriously associated with anticholinergic side effects, which include dry eyes, dry mouth, urinary retention, constipation, mydriasis and blurred vision. Ranitidine (D), and other H2-antagonists, are used to treat the reflux, not dry mouth, symptoms of Sjogren's.

Which of the following statements is correct regarding the classification of heart failure? Stage A refers to patients with symptoms at rest Stage B refers to patients with valvular dysfunction who are asymptomatic Stage C refers to patients with left ventricular dysfunction who are symptomatic with exertion Stage D refers to patients who are at risk of congestive heart failure but have no left ventricular dysfunction

Correct Answer ( C ) Explanation: Stage C describes patients with left ventricular dysfunction who are symptomatic with exertion. This is equivalent to the NYHC Class II and Class III. This includes about five million people in the United States. The most recent ACC/AHA classification for heart failure takes into account risk factors in addition to the presence of left ventricular dysfunction and symptoms. This classification complements the New York Heart Classification and considers recent advances in pharmacologic and non-pharmacologic approaches to evaluate and treat heart failure. This classification reliably allows the physician to follow patients as their heart failure progresses from one stage to the next, and offers a unique set of treatments appropriate to each stage. Stage A (A) describes patients who are at risk of heart failure but are asymptomatic and have no left ventricular dysfunction. More than 60 million people fall into this category and include those with coronary artery disease, hypertension, diabetes mellitus, and a family history of cardiomyopathy. Stage A is an additional classification that was not present in the prior NYHC. Stage B (B) describes patients with left ventricular dysfunction who are asymptomatic. This is equivalent to Class I of the NYHC and includes approximately ten million people in the United States. Stage D (D) describes patients with symptoms at rest. This is equivalent to Class IV of the NYHC. This includes approximately 200,000 people in the United States.

Question: What is the clinical diagnostic triad for Wernicke encephalopathy?

Question: What is the clinical diagnostic triad for Wernicke encephalopathy?

Which of the following fractures is most commonly non-operative in adults? Galeazzi Monteggia Radial head fracture Tibial plateau fracture

Correct Answer ( C ) Explanation: The majority of patients with a radial head fracture will not require operative management. Radial head fractures are typically caused by indirect trauma (e.g. fall on an outstretched hand). There are 4 types of radial head fractures ranging from the more common type I (undisplaced fracture) and type II (marginal fracture with minimal displacement) to the rarer type III (comminuted fracture) and type IV (fracture with dislocation). Type I and II injuries are usually treated with a sling and range of motion exercises. It is important to fully test the patient's range of motion. If the range of motion is limited by pain, the joint should be injected with an anesthetic agent (lidocaine, bupivicaine) and tested again. Continued limited range of motion in spite of pain control suggests the presence of entrapped fragments and orthopedic surgery may be required. Galeazzi (A) and Monteggia (B) fractures of the forearm have poor outcomes with nonsurgical management. The majority of tibial plateau fractures (D) also require surgery for acceptable functional outcomes.

A 5-week-old infant suffers from dyspnea and fluid overload but not cyanosis. Auscultation reveals a loud, holosystolic murmur at the left sternal border. You suspect a congenital cardiac defect. Echocardiography would most likely show which of the following abnormalities? Aorta is connected to the right ventricle Pulmonary artery is connected to the left ventricle Ventricular septal defect in the membranous portion Ventricular septal defect in the muscular portion

Correct Answer ( C ) Explanation: The most common congenital heart defect is a ventricular septal defect (VSD), one of the acyanotic congenital heart diseases. Defects occur in the ventricular septum, the "wall" between the right and left ventricle. This septum is comprised of an inferior muscular part and a superior membranous part. The majority of VSDs occur in the membranous part, near the atrioventricular node. A pansystolic murmur is nearly pathognomonic for VSD. Occurrence and severity of clinical manifestations depend on the size of the defect. Spontaneous closure occurs in the first six-months of life in up to 40% of affected infants. If present, symptoms may be mild or severe, but usually present as congestive heart failure due to a significant left-to-right intracardiac shunt with resultant pulmonary hypertension. Pharmacotherapy is recommended in these patients. However, surgical correction is typically necessary if the patient fails medical management or shows evidence of pulmonary hypertension (elevated pulmonary artery pressure). Transposition of the great arteries (A and B) is usually a cyanotic, not acyanotic, congenital cardiac defect in which the aorta is connected to the right ventricle and the pulmonary artery is connected to the left ventricle. It is a rare defect, affecting only 5% of all infants with congenital defects. If a VSD is concurrent, a loud pansystolic murmur is likely present, however, the infant would still have significant cyanosis. The presenting signs and symptoms suggest VSD, however, the majority of them occur in the membranous, not muscular (D), portion of the ventricular septum.

Which of the following is the initial treatment of choice in an overweight patient with moderate obstructive sleep apnea? Albuterol nebulizer 30 minutes prior to sleep Oral appliances Tracheostomy Weight loss and continuous positive airway pressure

Correct Answer ( D ) Explanation: Weight loss and continue positive airway pressure remains the most proven and effective therapy for obstructive sleep apnea. The continuous positive airway pressure acts as a pneumatic splint, holding the airway open. In addition, even a small reduction in body weight is associated with clinically significant improvements, and weight loss alone may be curative in some patients. Other lifestyle or conservative measures may include curbing alcohol intake before bedtime and avoiding a supine posture if obstructive sleep apnea is position dependent. Albuterol (A) use has not been shown to help in the management of sleep apnea. Tracheostomy (C), which bypasses the entire upper airway, is an effective treatment for obstructive sleep apnea, but it is invasive and should be reserved for patients with life-threatening arrhythmias or those who have failed conservative treatment. Oral appliances (B), or mandibular advancement devices, are used to increase upper airway caliber and are modestly effective for patients with mild to moderate obstructive sleep apnea.

Which of the following is the most likely etiologic agent of the rash pictured above? Coxsackievirus Group A Human herpes virus 6 Parvovirus B19 Rubella virus

Correct Answer ( C ) Explanation: The most commonly recognized manifestation of parvovirus B19 infection is erythema infectiosum, also known as fifth disease. This is a benign, self-limited childhood exanthem. After a two-week incubation period, patients often develop very mild URI-type symptoms followed in three to four days by the classic "slapped-cheek" rash that appears suddenly. It is believed that the development of the rash corresponds to the onset of immune response and sudden production of anti-B19 antibodies. Parvovirus B19 has also been associated with the development of severe disease, including a symmetric polyarthropathy, myocarditis, pericarditis, glomerulonephritis, and focal glomerulosclerosis. Primary infection in patients with sickle cell disease may lead to an aplastic crisis due to disruption of hematopoiesis. Nonimmune women who become infected during pregnancy are at risk of congenital infection that may lead to hydrops fetalis and fetal loss. Coxsackievirus Group A (A) is the etiologic agent associated with hand, foot, and mouth disease and causes painful blisters on the palms, soles, and posterior oropharynx. Human herpesvirus 6 (B) is one of the etiologic agents associated with roseola infantum (roseola), characterized by a nonpruritic erythematous rash that starts on the trunk and spreads to the arms and legs and is preceded by a high fever. Rubella virus (D) is the etiologic agent associated with rubella, also known as German measles, which is characterized by a light pink, lacy, pruritic rash that starts on the face, spreads rapidly to the trunk, and fades after three days.

What is the most common cause of croup? Adenovirus Haemophilus influenzae type b Parainfluenza virus Streptococcus spp.

Correct Answer ( C ) Explanation: The parainfluenza virus is the most common cause of croup. Croup is classically associated with a barking, seal-like cough and inspiratory stridor. Radiographs may show subglottic narrowing ("steeple sign") caused by edema. The typical age group is 6 months to 3 years, but the condition can be seen in children up to 5 years. The infection and inflammation are usually self-limiting, and conservative management is recommended. Evidence supports the routine use of corticosteroids in most children with croup. Intervention at an earlier phase of the illness reduces the severity of symptoms and the rates of return to a health care practitioner for additional medical attention, ED visits, and hospital admissions. Many children respond to a single, oral dose of dexamethasone. For those who do not tolerate the oral preparation, nebulized budesonide or intramuscular dexamethasone are reasonable alternatives. Mild disease can be treated with humidified oxygen. Moderate to severe disease should be treated with steroids and nebulized racemic epinephrine. Adenovirus (A) is a common cause of URIs, but less common than parainfluenza virus as a cause for croup. Haemophilus influenzae type b (B) is implicated in various infections such as otitis media and epiglottitis, but less common as a cause of croup. Streptococcus spp. (D) infection is not a common cause of croup.

A 15-year-old girl presents to the Emergency Department with complaints of vaginal pruritus and discharge. She denies sexual activity. Speculum examination reveals a homogenous, grey discharge with a pH > 4.5. A wet prep reveals stippled epithelial cells. What treatment is indicated? Azithromycin Ceftriaxone Metronidazole No treatment

Correct Answer ( C ) Explanation: The patient has classic findings of bacterial vaginosis. Bacterial vaginosis is not a sexually transmitted infection. Instead, it represents an overgrowth of vaginal anaerobic bacteria. Risk factors for bacterial vaginosis in all females include douching and menstruation. Sexually active women are at higher risk with increasing numbers of sexual partners, a new sexual partner, or if not using condoms. Gram stain may also aid in diagnosis. Vaginal secretions will reveal a paucity of gram positive rods (lactobacilli) and instead a predominance of gram negative or gram-variable rods and cocci. Culture is not recommended for diagnosis, as it will reveal normal vaginal flora but may not note the predominance of anaerobic bacteria. The treatment of choice for bacterial vaginosis is metronidazole. Treatment can be administered orally for seven days or intravaginally for five days in non-pregnant females, but only the oral form should be used in pregnant women. Oral tinidazole or intravaginal clindamycin are alternative regimens in non-pregnant women. Azithromycin (A) is the treatment of choice for chlamydial cervicitis. Chlamydia infection may cause vaginal discharge in females, but often the infection is asymptomatic. On speculum exam, the cervix is erythematous and friable. Ceftriaxone (B), a third generation cephalosporin, is the drug of choice for infections with Neisseria gonorrhoeae. Similar to chlamydial infection, gonorrheal infections may cause vaginal discharge, and, less commonly, may be asymptomatic. The cervix may also be erythematous and friable on speculum examination. No treatment (D) is inappropriate in a patient with symptoms and signs of bacterial vaginosis. Although bacterial vaginosis represents an overgrowth of normal anaerobic vaginal bacteria rather than an acquired infection, treatment is recommended. Adequate treatment is especially important for pregnant women, as bacterial vaginosis increases the risk of preterm delivery, chorioamnionitis, and endometritis.

A previously healthy 12-year-old girl presents with right ear pain for the past two days. On examination, there is erythema and edema of the external auditory canal with debris and crusted material visible in the external canal. She reports pain when the auricle is gently pressed. Which of the following pathogens is the most likely causative agent? Aspergillus Moraxella catarrhalis Pseudomonas aeruginosa Streptococcus pneumoniae

Correct Answer ( C ) Explanation: The patient has otitis externa. Otitis externa refers to infection and inflammation of the external ear. Factors which predispose to otitis externa include local trauma (often from scratching or attempted cerumen removal) and elevation of the local pH (frequent contact with water from swimming in pools or freshwater lakes). The most common organisms causing otitis externa are Pseudomonas aeruginosa, Staphylococcus aureus, Enterobacteriaceae, and Proteus species. Fungal species (like Aspergillus and Candida) cause otomycosis and are seen in immunosuppressed hosts, those with recent long-term antibiotic therapy, or in hot humid climates. The clinical presentation of otitis externa includes pruritus, pain, erythema, edema, and tenderness of the external auditory canal. Clear or purulent otorrhea may be present and crusting on the external canal is common. Infection may spread to the tragus, auricle, and periauricular soft tissues. Local lymph nodes may be inflamed. The treatment for otitis externa includes analgesia, cleansing of the external canal, and topical antimicrobials with or without steroids. Non-ototoxic topical antibiotics should be selected, particularly if swelling interferes with middle ear visualization and tympanic membrane rupture cannot be excluded. If edema of the external canal obstructs the lumen, an ear wick should be used to enhance distal delivery of the topical drops. Systemic antibiotics are not usually indicated, unless fever or periauricular extension is present. Aspergillus (A) and other fungal causes of otitis externa are usually seen in the immunocompromised or those with previous long-term antibiotic therapy. Moraxella catarrhalis (B) and Streptococcus pneumonia (D) are common causes of otitis media, not otitis externa.

A 75-year-old woman with a history of hypertension presents to the ED with right upper quadrant pain that has been gradually worsening over the past day. Her vital signs are: T 103°F, BP 100/60, HR 100, RR 22, and oxygen saturation 97% on room air. Physical exam reveals scleral icterus and right upper quadrant tenderness without rebound or guarding. A bedside right upper quadrant ultrasound demonstrates a common bile duct measuring 1 cm. Laboratory results are pending. What is the definitive management of this condition? Broad spectrum antibiotics Cholecystectomy Endoscopic retrograde cholangiopancreatography Percutaneous transhepatic cholangiography

Correct Answer ( C ) Explanation: The patient presents with severe (suppurative) ascending cholangitis, and requires biliary decompression with endoscopic retrograde cholangiopancreatography (ERCP). Ascending (or acute) cholangitis is a bacterial infection of the biliary system and is most frequently associated with common bile duct stones and obstruction. The disease classically presents with Charcot's triad (fever, jaundice, and right upper quadrant pain); however, only 50-75% of patients present classically, so clinicians must maintain a high index of suspicion for this life-threatening condition. While administration of broad spectrum antibiotics is appropriate, those with signs of severe disease—including persistent abdominal pain, hypotension despite adequate fluid resuscitation, fever greater than 102°F, and confusion—warrant urgent ERCP. Seventy to eighty percent of patients with ascending cholangitis will respond to broad spectrum antibiotics (A) such as piperacillin-tazobactam. However, definitive management of this condition consists of biliary decompression. Cholecystectomy (B) may be required after disease resolution to prevent recurrent episodes, but is no longer part of the definitive management of ascending cholangitis due to its high associated mortality. Percutaneous transhepatic cholangiography (D) can be considered if ERCP is unavailable, unsuccessful, or contraindicated.

Question: What life-threatening arrhythmia can QT prolongation progress to?

Question: What life-threatening arrhythmia can QT prolongation progress to?

A 35-year-old man with a history of alcohol abuse presents to your office with complaints of a 3-day history of abdominal pain and vomiting after a night of binge drinking. He has not eaten in days. Laboratory values show an elevated anion gap, ketonemia and normal glucose levels. In addition to thiamine, what other treatment should be provided for this patient? Bicarbonate and insulin Glucagon and hydrocortisone Normal saline and glucose Pyridoxine

Correct Answer ( C ) Explanation: This patient is suffering from alcoholic ketoacidosis, which is defined by the clinical triad of ketones in the blood or urine, an elevated anion gap and a normal glucose level. Classically, these patients are chronic alcoholics and in a state of starvation. Blood alcohol may be undetectable and the patient may be hypoglycemic. Patients typically have high osmol and anion gaps. Treatment of alcoholic ketoacidosis includes vigorous volume repletion with normal saline, along with administration of thiamine and glucose. Alcohol appears to significantly increase the amount of thiamine required to treat the patient successfully compared with other individuals in whom thiamine deficiency has been due to starvation. Thiamine is administered initially before giving glucose and normal saline to the patient. Administering dextrose to an individual in a thiamine-deficient state exacerbates the process of cell death and can worsen the patient. Bicarbonate and insulin (A) is administered to patients in severe diabetic ketoacidosis with a pH less than 7. Levels of glucagon and hydrocortisone (B) are typically elevated in patients with alcoholic ketoacidosis. The body is in a state of starvation, which results in hypoglycemia triggering the release of glucagon. The body is undergoing a stressful state causing the release of hydrocortisone. Pyridoxine (D), also known as vitamin B6, is not used for the treatment of alcoholic ketoacidosis. Common uses of this vitamin include anti-emetic in pregnancy and as an isoniazid adjunct in the treatment of tuberculosis.

A 44-year-old woman with no past medical history presents with a red eye. She denies pain or change in vision. What management is indicated? Check a complete blood count and coagulation studies Emergent ophthalmologic consultation Reassurance and follow up Topical antibiotics

Correct Answer ( C ) Explanation: This patient presents with a subconjunctival hemorrhage and should be reassured and given follow up with an opthalmologist or primary care provider. Subconjunctival hemorrhage results from rupture of small subconjunctival blood vessels. It is thought to occur from trauma or Valsalva maneuvers (e.g. coughing) although patients often have no memory of an inciting event. There should not be any pain, change in visual acuity or photophobia associated with this finding. Cosmetic appearance is usually the most prevalent concern. These lesions are benign and resolve in 2-3 weeks. In patients with a simple subconjunctival hemorrhage, labs are not required (A). Emergent ophthalmologic consultation (B) is not required as no intervention is required. Topical antibiotics (D) do not aid in healing or prevent infection.

A 3-year-old boy presents with his mother after he tripped and sustained an injury to his mouth. The mother has his avulsed tooth soaking in a container of milk. Which of the following is true concerning the management of dental avulsions? A primary central incisor should be reimplanted A primary tooth avulsion often leads to a cosmetic deformity of the secondary tooth Avulsed teeth should be held by their root to avoid contaminating the crown Milk is an acceptable storage medium for an avulsed tooth

Correct Answer ( D ) Explanation: A dental avulsion occurs when a tooth is completely removed from its socket. Management of recovered avulsed teeth depends on the age of the patient and the length of time that the tooth has been absent from the oral cavity. Avulsed primary teeth in a pediatric patient age 6 months to 6 years are not replaced in the socket. The best environment for an avulsed tooth is in its socket. The greatest success for reimplantation is 30 minutes. The worst situation is to allow the tooth to be transported in a dry medium. Storage in plain water is not much better. Although saliva is a reasonable storage medium, milk is preferable because of its osmolarity and essential ion concentration of calcium and magnesium. The best storage and transport medium is Hanks solution, a balanced pH cell culture medium. Reimplanting primary teeth (A and B) ankylose or fuse to the bone so that although the dentofacial complex grows downward and forward, the reimplantation site does not. There also may be interference with the eruption of the permanent tooth. Hence primary teeth should not be reimplanted. Avulsed teeth should be handled only by the crown (C) so that the remaining periodontal ligament fibers are not damaged.

A 30-year-old woman presents with abnormal menstrual bleeding. Her history and physical are unrevealing. Other than laboratory testing, which of the following is the next best step in evaluating this patient? Dilatation and curettage Endometrial biopsy Hysteroscopy Pelvic ultrasonography

Correct Answer ( D ) Explanation: Determining the source of abnormal uterine bleeding is a difficult task when caring for gynecologic patients. This evaluation begins with a detailed history and physical followed by laboratory tests and pelvic ultrasound. Common initial laboratory tests include a complete blood count, serum prolactin level, serum ß-hCG and serum thyroid stimulating hormone level. Pelvic ultrasonography may reveal uterine lining abnormalities, including irregularities, polyps, fibroids and masses. The results of the above will guide the clinician down a non-gynecologic versus gynecologic pathway. If a gynecologic source is suggested, endometrial biopsy (B), which can be accomplished via many different curette procedures, is then recommended. If the source of bleeding is still not found, hysteroscopy (C), which allows direct visualization of the uterine cavity, is then recommended. If the diagnosis is still unclear, the patient may be sent for dilatation and curettage (A). This procedure's sedation allows for muscular relaxation which may increase diagnostic yield.

A 76-year-old man presents to the clinic with the chief complaint of "poor vision." He is an otherwise healthy man, but relays a strong family history of macular degeneration. Which of the following symptoms would be most consistent with a diagnosis of macular degeneration? A "curtain" spreading across the field of vision Blurred vision with glares noticed while driving at night Gradual bilateral peripheral vision loss Progressive deterioration of central vision

Correct Answer ( D ) Explanation: Macular degeneration results in a loss of central vision that may be acute or progressive. Macular degeneration is typically a disease of older patients, and is the most common cause of permanent vision loss in this population. There is a white, female predominance. Family history and cigarette smoking are also risk factors. Age-related macular degeneration may be classified as either atrophic ("dry") or neovascular ("wet"). Both types are progressive and usually bilateral. However, dry macular generation is usually moderate in severity, and related to the atrophy and degeneration of the outer retina and retinal pigment epithelium. In contrast, wet macular degeneration typically causes more severe and rapid vision loss, as it is due to the accumulation of serous fluid, fibrosis, and hemorrhage that occur when choroidal new vessels form between the retinal pigment epithelium and Bruch membrane. Treatment depends on the type of macular degeneration. There is no specific treatment for dry macular degeneration; low vision aids should be recommended when necessary. Wet macular degeneration may be managed with monthly intravitreal injections of inhibitors of vascular endothelial growth factors. Research has also shown that oral treatment with vitamins A, C, and E, in addition to zinc and copper, may slow progression. Ophthalmology referral is necessary for appropriate management of this condition. A "curtain" spreading across the field of vision (A) is the stereotypical complaint of a patient with retinal detachment. This is typically an acute condition with visual distortions presenting and progressing over hours to days, and should be referred urgently to ophthalmology. Blurred vision with glares noticed while driving at night (B) is a common complaint of patients who have cataracts. While this is also commonly a progressive disease of older patients, central vision is not predominantly affected. Rather, the entire visual field appears blurred. Gradual bilateral peripheral vision loss (C) is the presenting symptom for chronic glaucoma. This "tunnel vision" usually allows for some preservation of central vision with intact visual acuities, unlike macular degeneration.

Which of the following is the most common form of liver disease in the United States? Hepatitis A Hepatitis B Hepatitis C Nonalcoholic fatty liver disease

Correct Answer ( D ) Explanation: Nonalcoholic fatty liver (NAFL) disease is, more or less, a benign condition that has become increasingly common in the United States and Western Europe as weight gain and obesity have become more common. It is now the most common cause of liver disorder in the United States and other Western industrialized countries. In fatty liver, the liver functions normally and looks normal under the microscope, except for accumulations of fat within cells. NAFL is often detected when imaging tests of the abdomen are obtained for other reasons (e.g. an ultrasound being done to look for gallstones). Liver blood tests are either normal, or slightly increased. The diagnosis may be confirmed with a right upper quadrant ultrasound examination. If the liver blood tests are significantly elevated or if there are other signs of liver disease (e.g. jaundice), then a liver biopsy may be recommended to look for other problems such as nonalcoholic steatohepatitis (NASH). Aside from losing weight, there is no other treatment, though people with liver disease should be vaccinated against hepatitis A and B if they are not already immune. Hepatitis A (A) is spread via the fecal-oral route directly via person-to-person contact or indirectly through the ingestion of contaminated water or food (raw or undercooked shellfish). There is no carrier state, and it does not cause chronic liver disease. Hepatitis B (B) is spread primarily by the percutaneous route but also may be transmitted by intimate contact via bodily fluids. Fulminant hepatic failure develops in 1% of cases. The carrier state is associated with an increased risk for developing hepatocellular carcinoma. Hepatitis C (C) is the most common form of viral hepatitis in the United States. Prior to 1992, most cases of hepatitis C occurred in association with transfusion of blood and blood products. During this time, hepatitis C was referred to as post-transfusion hepatitis. Currently, intravenous and intranasal drug abuse accounts for the increase in cases.

A 24-year-old man presents with complaints of the "flu" for the past five weeks. He complains of having a fever, cough, and runny nose about one month ago that has since resolved. Now he complains of a chronic dry cough that "comes in fits" and seems to be worse at night when he is often awakened from sleep. He denies fevers, chills, rhinorrhea, or nasal congestion. He saw his primary care physician two weeks ago who obtained a chest X-ray (negative) and diagnosed him with a viral infection. Your physical exam reveals a well-appearing male who is in no acute distress. The cardiopulmonary exam is within normal limits and a repeat chest X-ray in the ED is unremarkable. Which of the following is the most likely diagnosis? Aspirated foreign body Congestive heart failure Cough-variant asthma Pertussis

Correct Answer ( D ) Explanation: Pertussis (whooping cough) primarily affects children younger than 10 years old, but the number and proportion of cases involving adults are increasing. Pertussis is difficult to diagnose. It typically begins with a catarrhal stage (rhinorrhea and mild cough), which can last up to two weeks. This is followed by paroxysms of an irritating cough. The repeated violent coughs consist of short expiratory bursts followed by an inspiratory gasp, or "whoop," that gives the disease its characteristic name. However, the whoop is often absent in adults and in infants younger than six months old. Although the diagnosis is based primarily on the characteristic clinical features, definitive diagnosis is made through the culture of nasopharyngeal aspirate on specific culture medium. Direct fluorescent antibody (DFA) and polymerase chain reaction (PCR) can be used for confirmation. Click below for Audio of the Pertussis "whoop" 00:0000:00 An aspirated foreign body (A) most commonly occurs in children younger than three years, elderly, alcoholics, and those with central nervous system disease. Clinically, patients may present with inspiratory stridor and wheezing. Symptoms should be persistent rather than come in "fits" or be worse at night as with the patient in the above scenario. Congestive heart failure (B) would be an uncommon diagnosis in a 24-year-old. The condition is associated with coughing and orthopnea. The chest radiograph generally demonstrates an enlarged cardiac silhouette with increased pulmonary vasculature congestion. Cough-variant asthma (C) is a type of asthma in which the main symptom is a dry, nonproductive cough and is not associated with classic asthma symptoms such as wheezing or shortness of breath. This condition is similar to pertussis in that both are associated with a chronic cough, but pertussis is also associated with a catarrhal period (coryza, mild fever, nonproductive cough).

A 40-year-old woman presents with weakness, fatigue, nausea, and diarrhea. Physical exam reveals orthostatic hypotension and axillary fold hyperpigmentation. You obtain a random cortisol level of 2.4 µg/dL. Which of the following laboratory abnormalities would you expect to find in this patient? Hypercalcemia Hypermagnesemia Hypokalemia Hyponatremia

Correct Answer ( D ) Explanation: Primary adrenal insufficiency (Addison's disease), or hypocortisolism, is most commonly caused by autoimmune destruction of the adrenal cortex. Nonspecific symptoms of hypocortisolism include weakness, fatigue, weight loss, anorexia, orthostasis, and listlessness. The most specific sign of primary adrenal insufficiency is hyperpigmentation, typically of the mucous membranes, axillary folds, and nipples. Another specific symptom of primary adrenal insufficiency is salt craving. Because the adrenal cortex is damaged, aldosterone levels are also affected. A decrease in aldosterone will lead to less renal sodium reabsorption and less renal potassium excretion, with subsequent hyponatremia and hyperkalemia. A net loss of sodium to the urine may lead to polyuria and hypovolemia. Hyponatremia associated with adrenal insufficiency may cause seizures, delirium, coma, or death. Hyperkalemia, not hypokalemia (C) is seen in primary adrenal insufficiency. Parathyroid hormone, not cortisol, is responsible for maintaining calcium homeostasis (A). Cortisol has no effect on magnesium (B) levels.

A 26-year-old woman at 32 weeks gestation complains of swelling and discomfort in her right leg. Ultrasound of the leg reveals a non-compressible vein. Which of the following treatments is contraindicated? Inferior vena cava filter Low molecular weight heparin Unfractionated heparin Warfarin

Correct Answer ( D ) Explanation: The patient has a deep venous thrombosis. Pregnancy induces a hypercoagulable state, therefore pregnant patients are 5-10 times more likely to develop venous thromboembolism (VTE) than their non-pregnant counterparts. The causes of hypercoagulability in pregnancy include: increased levels of clotting factors, increased platelet and fibrin activation, and decreased fibrinolytic activity. Additionally, venous distention of pregnancy may contribute to venous stasis and further increases the risk of VTE. The first-line treatment of VTE in pregnancy includes anticoagulation with heparin or low-molecular weight heparin. These agents do not cross the placental barrier and are considered safe in pregnancy. Warfarin is a known teratogen, which causes fetal developmental abnormalities (midface hypoplasia, stippled chondral calcification, scoliosis, and short proximal limbs) and is contraindicated in pregnancy. There is little data on the use of thrombolytics, such as tissue plasminogen activator (tPA) in pregnancy, however thrombolytics are recommended in life-threatening cases, (massive pulmonary embolism). Embolectomy can also be considered in severe cases. An inferior vena cava filter (A) is an appropriate treatment option in pregnancy when anticoagulation is contraindicated or in those who have recurrent events while on anticoagulation. Low molecular weight heparin (B) and unfractionated heparin (C) are appropriate treatments for venous thromboembolism in pregnancy.

A 43-year-old woman presents with right upper quadrant abdominal pain for 3 weeks. She states that she intermittently gets sharp pain that occurs after eating and is associated with nausea and occasionally, vomiting. The pain lasts for 10-15 minutes and then spontaneously improves. Currently, she has no pain. Her vitals and blood work are normal. A right upper quadrant ultrasound is shown above. What management is indicated? Administer antibiotics and admit for observation Admit patient for cholecystectomy Obtain CT scan of the abdomen and pelvis Referral for surgical consultation and pain medication as needed

Correct Answer ( D ) Explanation: This patient presents with biliary colic as a result of cholelithiasis (gall stones). Cholelithiasis is a common disorder affecting 20% of women and 8% of men. Cholelithiasis usually presents with intermittent right upper quadrant (RUQ) or epigastric pain which may radiate to the base of the right scapula or shoulder. The pain is often associated with nausea and vomiting. Attacks are usually self-limited and often occur after eating. The theory is that the gallbladder contracts in response to food and contraction against the stones causes pain. Laboratory studies are usually non-contributory but can be useful in ruling out common duct obstruction (elevated bilirubin and alkaline phosphatase) and pancreatitis (elevated lipase) which can occur when gallstones move out of the gallbladder. Plain radiographs are not helpful in diagnosis as only 10% of stones have enough calcium to be visualized. Ultrasound, on the other hand, is rapid and highly sensitive in the diagnosis of cholelithiasis and cholecystitis. In patients who present with typical symptoms of biliary colic with an ultrasound supporting this diagnosis, management should focus on supportive care (repleting fluids and electrolytes lost in vomiting), pain control, and ruling out other causes of abdominal pain. Once the symptoms are managed, patients can be discharged with appropriate follow up. Advising the avoidance of fatty foods and alcohol can reduce the occurrence of symptoms. The definitive management of cholelithiasis is cholecystectomy. The decision for cholecystectomy (B) can be made with outpatient surgical consultation in patients with cholelithiasis. CT scan of the abdomen and pelvis (C) is useful in ruling out other causes of abdominal pain but is less sensitive than ultrasound for finding cholelithiais and cholecystitis. Although there is little evidence for the role of bacterial infection in cholecystitis, antibiotics (A) are recommended but are not necessary in the treatment of cholelithiasis. Admission (A and B) is also not necessary as long as the patient with cholelithiasis has adequate pain control.

A 24-year-old woman presents for initial evaluation of cyclical symptoms of irritability, painful bloating and depression. These symptoms occur regularly 4-5 days prior to the beginning of menstruation. During these few days, this patient typically has to miss work due to the "awful pain and mental clouding." Which of the following treatment options do you recommend as first-line therapy? Anticholinergics Hysterectomy Oral contraceptive pills Selective serotonin reuptake inhibitors

Correct Answer ( D ) Explanation: Treatment of mild premenstrual syndrome symptoms, those considered to not alter daily socioeconomic function, includes stress reduction techniques and regular exercise. This patient reports historical points which reveal moderate to severe symptomatology. In women whose daily function is altered due to the cyclical, luteal phase symptoms of premenstrual syndrome, selective serotonin reuptake inhibitors (SSRIs) are recommend as first-line therapy. Options include sertraline or fluoxetine, administered on a daily basis or only during the second half (luteal phase) of the patient's menstrual cycle. Anticholinergics (A) block central and peripheral nervous system acetylcholine transmission. Their common side effects of confusion, irritability, dysphoria, memory difficulty and poor concentration would not be recommended in a patient who is already cognitively and vocationally impaired. Surgery, consisting of hysterectomy (B) with bilateral oophorectomy, is reserved for rare refractory cases of severe disabling premenstrual syndrome symptoms. Oral contraceptive pills (OCPs) (C) are recommended as second line therapy in those women who cannot tolerate the side effects of, or do not respond to, SSRIs.


संबंधित स्टडी सेट्स

Chorioamnionitis (Amnionitis, Intra-Amniotic Infection)

View Set

Ch 18 Reading/Concept, Astro Chapters 15-19, Astro Final, 16, Pre-Lecture Chapter 17 + Visual Quiz, ASTR FINAL, Astronomy Ch. 17, ASTR=Chapter 18 Visual Quiz, Module 13: Exploring Our Galaxy: The Milky Way, Chapter 18 : Astronomy Reading Quiz

View Set